Mathematical and Physical Journal
for High Schools
Issued by the MATFUND Foundation
Already signed up?
New to KöMaL?

Problem A. 837. (November 2022)

A. 837. Let all the edges of tetrahedron \(\displaystyle A_1A_2A_3A_4\) be tangent to sphere \(\displaystyle S\). Let \(\displaystyle a_i\) denote the length of the tangent from \(\displaystyle A_i\) to \(\displaystyle S\). Prove that

\(\displaystyle \bigg(\sum_{i=1}^4 \frac 1{a_i}\bigg)^{\!\!2}> 2\bigg(\sum_{i=1}^4 \frac1{a_i^2}\bigg). \)

Submitted by Viktor Vígh, Szeged

(7 pont)

Deadline expired on December 12, 2022.


We will use Descartes' theorem: the radii of every four mutually tangent circles satisfy the following:

\(\displaystyle \left(\frac{1}{r_1}+\frac{1}{r_2}+\frac{1}{r_3}+\frac{1}{r_4}\right)^2=2\left(\frac{1}{r_1^2}+\frac{1}{r_2^2}+\frac{1}{r_3^2}+\frac{1}{r_4^2}\right).\)

Let \(\displaystyle S_i\) denote the sphere with center \(\displaystyle A_i\) and radius \(\displaystyle a_i\). Note that these spheres are mutually tangent to each other. Without loss of generality we can assume that \(\displaystyle a_1 \geq a_2 \geq a_3 \geq a_4\). Let's project these spheres orthogonally onto the plane of triangle \(\displaystyle A_1A_2A_3\). Let \(\displaystyle C_i\) denote the projection of \(\displaystyle S_i\) (which is a circle). The radii of these circles are equal to the radii of the spheres. Circles \(\displaystyle C_1\), \(\displaystyle C_2\) and \(\displaystyle C_3\) are still mutually externally tangent to each other, because we projected them onto the plane of their centers. \(\displaystyle C_4\) has to has a common point with all three circles. Also, it is not possible that \(\displaystyle C_4\) is tangent to all three circles, since the spheres would have to have the same points of tangency, but then \(\displaystyle A_4\) would have to be in the plane of the other three points.

Let \(\displaystyle k_i=\frac{1}{a_i}\) for \(\displaystyle 1 \leq i \leq 4\). Then \(\displaystyle k_1 \leq k_2 \leq k_3 \leq k_4\). Using these notations the claim we have to prove becomes

\(\displaystyle 2(k_1^2+k_2^2+k_3^2+k_4^2)<(k_1+k_2+k_3+k_4)^2.\)

Let us rearrange it for \(\displaystyle k_4\):

\(\displaystyle 0<-k_4^2+2k_4(k_1+k_2+k_3)+((k_1+k_2+k_3)^2-2(k_1^2+k_2^2+k_3^2)).\)

This is a quadratic equation for \(\displaystyle k_4\) the roots of which are

\(\displaystyle x_{1,2}=k_1+k_2+k_3 \pm 2\sqrt{k_1k_2+k_1k_3+k_2k_3}.\)

Thus we have to prove that \(\displaystyle x_1<k_4<x_2\).

Let \(\displaystyle C\) be the circle of smallest radius that has a point in common with circles \(\displaystyle C_1, C_2\) and \(\displaystyle C_3\). Clearly, this is a reasonable definition, there exists such a circle, namely the one which is in the region bounded by the circles \(\displaystyle C_1\), \(\displaystyle C_2\) and \(\displaystyle C_3\) and which touches all three circles (from the outside). Then, by Descartes' theorem

\(\displaystyle k=k_1+k_2+k_3+ 2\sqrt{k_1k_2+k_1k_3+k_2k_3},\)

where \(\displaystyle k\) is the reciprocal of the radius of the circle \(\displaystyle C\). Since the circle \(\displaystyle C_4\) has a point in common with the circles \(\displaystyle C_1\), \(\displaystyle C_2\) and \(\displaystyle C_3\), and does not touch all three, \(\displaystyle k_4<k=x_2\).

It remains only to prove that \(\displaystyle x_1<k_4\), i.e.

\(\displaystyle k_1+k_2+k_3- 2\sqrt{k_1k_2+k_1k_3+k_2k_3}<k_4.\)

We prove the stronger claim

\(\displaystyle k_1+k_2+k_3- 2\sqrt{k_1k_2+k_1k_3+k_2k_3}<k_3.\)

This is equivalent to

\(\displaystyle k_1^2+2k_1k_2+k^2<4(k_1k_2+k_1k_3+k_2k_3),\)

which is indeed satisfied, since \(\displaystyle k_1 \leq k_2 \leq k_3\), so \(\displaystyle 2k_1k_2<4k_1k_2\), \(\displaystyle k_1^2<4k_1k_3\) and \(\displaystyle k_2^2<4k_2k_3\). This completes the proof.


Statistics:

9 students sent a solution.
7 points:Diaconescu Tashi, Lovas Márton, Nádor Benedek, Seres-Szabó Márton, Sztranyák Gabriella, Wiener Anna.
5 points:1 student.
1 point:1 student.
0 point:1 student.

Problems in Mathematics of KöMaL, November 2022